ChaseDream

标题: lulu's备考日记之CR [打印本页]

作者: lululuna    时间: 2015-6-28 21:33
标题: lulu's备考日记之CR
CR备考日记

6/28

7/28 Review

65.(35041-!-item-!-188;#058&007595) Source:prep07-9
Sales of telephones have increased dramatically over the lastyear. In order totake advantage of this increase, Mammoth Industries plans to expand production of its own model of telephone, while continuing its already very extensiveadvertising of this product.
Which of the following, if true, provides most support for the view that Mammoth Industries cannot increase its sales of telephones byadopting the plan outlined above?
(A) Although it sells all of the telephones thatit produces, Mammoth Industries' share of all telephone sales has declined overthe last year.
(B) Mammoth Industries' average inventory oftelephones awaiting shipment to retailers has declined slightly over the lastyear.
(C) Advertising has made the brand name of MammothIndustries' telephones widely known, but few consumers know that MammothIndustries owns this brand.
(D) Mammoth Industries' telephone is one of threebrands of telephone that have together accounted for the bulk of the lastyear's increase in sales.
(E)Despite a slight decline in the retail price, sales of Mammoth Industries'telephones have fallen in the last year.

P: adpot the plan
    (expand production+ continue already extensive ads)
C: X increase sale


Answer:E
解析:
正确答案需要否定plan,因为Q: Mammoth Industries cannot increase its sales of telephones by adopting the plan outlined above
把不能increase sale的原因,归咎于Plan, 那么正确答案需要说出plan不好的地方
* special attetion tip: 逻辑中名词前面的modifier一定程度上表达了倾向,例如此题,already very extensive advertising of this product
E选项,尽管过去一年中电话的ads已经很intensive了,但是去年的销售还是下降了,那么continue ads(as part of the plan is useless)
A,sell all products,那么 expand production就可以继续sell, support the plan;
B, invetory 等待发货减少了,和销售量无关,说不定是物流进步了呢;
C,brand name irrelevant to plan above;
D,M的电话是去年卖的最好的三个品牌,那可能证明already extensive ads 是有效的,可以continue,不能否定plan;
自己不理解的原因:看选项就用过去 不等于 将来,排除了E项(谨记,千万不能用General rule来排除任何选项!case by case study


作者: lululuna    时间: 2015-6-28 22:54
6/28
7/28review

Manufacturers sometimes discount the price of aproduct to retailers for a promotion period when the product is advertised to consumers.  Such promotions often resultin a dramatic increase in amount of product sold by the manufacturers to retailers.  Nevertheless, the manufacturers could often make more profit by not holding the promotions.

Which of the following, if true, most strongly supports the claim above about the manufacturers' profit?

(A) The amount of discount generally offered by manufacturers to retailers is carefully calculated to represent the minimum needed to draw consumers' attention to the product.
(B) For many consumer products the period ofa dvertising discounted prices to consumers is about a week, not sufficientlylong for consumers to become used to the sale price.
(C) For products that are not newly introduced,the purpose of such promotions is to keep the products in the minds of consumers and to attract consumers who are currently using competing products.
(D) During such a promotion retailers tend toaccumulate in their warehouses inventory bought at discount; they then sell much of it later at their regular price.
(E) If a manufacturer fails to offer such promotionsbut its competitor offers them, that competitor will tend to attract consumers away from the manufacturer's product.

P:  X hold the discount promotions
C:  make more profit

OA:E
本题涉及三方:Manufacture, Retailer,  Consumer
题目只有提及了M对R采取的策略:discount prc for product; 虽然Intenions是:ads to C,但未提及R会对C采取何种措施;
所以M能否实现ads to C, 需要看R会对C 采取何种措施;尽管此题问的不是这个,但解题的思路是一样的。
*special tip: 涉及Manufacture, Retailer,  Consumer三方时,要注意跳跃性逻辑关系
这道题做的时间有点长,还是没有理清涉及的各方之间的关系


7/12补充题
Prep2012-Pack1-CR-031       VCR000875     Medium
Which of the followingmost logically completes the passage below?
One name-brand cereal manufacturer is about to reduce wholesale prices for its cereals by 20 percent because consumers have been switching from its cereals to cheaper store brands.The success of this strategy relies on the assumption that supermarkets will pass on all of the savings by lowering the prices they charge consumers for the manufacturer's cereals. Although supermarkets usually pass on such savings, in this case it is likely that supermarkets not do so because            .

A. several other name-brand cereal manufacturers are about to reduce the wholesale prices of their cereals
B. the average price per box of name-brand cereals has increased significantly in the last 10years
C. total annual sales of cereal—including both name-brand and store-brand cereals—have increased over the past 10 years
D.  supermarkets make far more profit on sales of store-brand cereals than on sales of name-brand cereals
E. the current prices of the manufacturer's cereals are comparable to the pricesof name-brand cerealsproduced by other cereal manufacturers

OA: D
此题涉及三方,name-brand manufacturer(manufacturer), supermarket(retailer), customer
似乎这类题目涉及到 manufacturer想要对customer采取行动总是要涉及到retailer,那么中间的retailer采取的策略就至关重要;
属于A=>B=>C; A直接想要作用于C,那么必须有ASSUMPTION就是B会按照A的plan或者既定意愿去做,否则不逻辑链就断了;



作者: lululuna    时间: 2015-6-30 22:20
6/30
7/28review

Prep2-24(28903-!-item-!-188;#058&004064)

Although exposure to asbestos is the primary cause of mesothelioma, a slow-developing cancer, researchers believe that infection by the SV40 virus is a contributing cause, since in the United States 60 percent of tissue samples from mesotheliomas, but none from healthy tissue, contain SV40. SV40 is a monkey virus; however, in 1960 some polio vaccine was contaminated with the virus. Researchers hypothesize that this vaccine was the source of the virus found in mesotheliomas decades later.

Which of the following, if true, most strongly supports the researchers' hypothesis?
(A) SV40 is widely used as a research tool in cancer laboratories.
(B) Changes in the technique of manufacturing the vaccine now prevent contamination with SV40.
(C) Recently discovered samples of the vaccine dating from 1960 still show traces of the virus.
(D) In a small percentage of cases of mesothelioma, there is no history of exposure to asbestos.
(E) In Finland, where the polio vaccine was never contaminated, samples from mesotheliomas do not contain SV40.

OA:E
P: 60% M tissue contain SV40, X M doesn‘t contain SV40
C:   SV40 is a cause of M

P: in 1960, V is contaminated with SV40
C:   V is the source of SV40 in M=> V cause M

类型:无因无果(逆否命题)型加强

Strengthen, 正确的加强,需要将V和M 之间的关系明确化,
文中用的是假设了这个结果,而且事实上题干上也没有将V和M之间的关系直接家里起来,而是通过了SV40这个中介。

E项,逆否命题,无因无果型加强
C项, V和SV40之间至今仍然有关系,并不能说明V cause M

http://forum.chasedream.com/foru ... t=prep2%2B24&page=2
#11
One way to strengthen an argument is to show that when the cause does not occur, the effect does not occur. In our case, this would lead to the following equation:
NO contamination of virus in polo vaccine --> NO virus found in mesotheliomas.  That is exactly what is said in answer (E).

#16
题目中想找A这种现象的原因。因为A有C,所以试图建立A与C的关系。想通过桥梁B来建立。给出的信息是 C与B在1960年有密切关系。问你哪一个support。 C中讲的是B和C现在还有关系。这个 对于AC关系的建立毫无意义。E,是逆否命题。
用这个题里的东西来代。
某种疾病M的发生因为病毒S。因为病毒S感染了疫苗V在1960年。假设 疫苗V是疾病M的源头。C中说 现在 病毒S还感染了疫苗V。这个选项吧已知的条件加强了,但是对于V和M是啥关系只字未提。

special attention tip:
1)此题和自己这个帖子#2一样之处在于涉及了三个方面,加强和削弱的环节上有两层,重点应该关注在问的环节上,在对应的环节去加强;
2)对于无因无果,逆否命题型加强把握不好,比较难抓到,总是当成无关选项来排除,这个要特别注意。
   

7/4补充
sdcar2010
http://forum.chasedream.com/thread-569879-1-1.html
To find these jumps, look for any concepts that are discussed only once. There are usually two. The correct answer usually fills in the gap in the argument by linking those two concepts together. Alternatively, slowly read each word of the conclusion. Which phrase was not used in the premise? That word or phrase will likely reveal where the argument jumped to the conclusion: at the gap!


作者: lululuna    时间: 2015-6-30 22:49
6/30
prepCR27.(30565-!-item-!-188;#058&005567)
Among people who experience migraine headaches, some experience what doctors call "common" migraines, whereas others experience "classical" migraines.Siblings and spouses of common migraine sufferers are themselves twice as likely as the general population to experience common migraines.Siblings of classical migraine sufferers are four times more likely than the general population to experience classical migraines, whereas spouses of classical migraine sufferers are no more likely than the general population to experience such headaches.

The information above provides the most support for which of the following hypotheses?

(A) Susceptibility to classical migraines is more dependent on hereditary factors than is susceptibility to common migraines.
(B) Unmarried adults are more likely to suffer from classical migraines than they are to suffer from common migraines.
(C) People who do not experience migraine headaches are unlikely to have spouses who are migraine headache sufferers.
(D) Children of people who suffer from common migraines are not more likely than the general population to experience a common migraine.
(E) Between one-quarter and one-half of the general population suffer from either common or classical migraine headaches.

Common M: Sibling+ Spouse twice more than general
Classical M: Sibling 4 times more than general; Spouse no more than general

==》A

这题没什么问题,读懂了读仔细就可以,列在这儿给自己警示一下!不要慌,不要瞎!




作者: lululuna    时间: 2015-7-2 01:06
07/01
Prep2012-Pack1-CR-017       VCR007918     Easy
A physically active lifestyle has been shown to help increase longevity. In the Wistar region ofBellaria, the averageage at death is considerably higher than in any other part of the country.Wistar is the only mountainous part of Bellaria. A mountainous terrain makes even such basic activities as walking relatively strenuous; it essentially imposes a physically active lifestyle on people.Clearly, this circumstance explains the long lives of people in Wistar.

Which of the following, true, most seriously weakens the argument?

A.  In Bellaria all medical expenses are paid by the government, so thatpersonal income does not affect the quality of healthcare a person receives.
B.  People living in Wistar donot have a significantly better dietthan people livingin other parts of Bellaria.
C.  Many people who live in the Wistar regionhave moved there inmiddle age or upon retirement
D. The many opportunities for hiking, skiing, and other outdoor activities that Wistar’s mountains offer make it a favorite destination for vacationing Bellarians.
E.  Per capita spending on recreational activities is no higher in Wistar than it is in other regionsof Bellaria.

  OA:C

P: active lifestyle(mountainous terrain)  in W
C: longevity

类别:selecting bias型 削弱

A 整个B地区的医保和个人收入,无关
B W的人没有比B区内其他地方的人吃的更健康 ,加强
C, 许多住在W区的人时中年或者退休时候搬来的
     1)本身年龄大,拉大了平均年龄;
     2)后来搬来的,并不是这种长期的lifestyle使得他们长寿;可能他们本来就长寿,才能熬到退休( selecting bias?)
     我更prefer 2)的解释,1)有点牵强,与前提无关联;2)的优点在于削弱了前提
    错的原因:*focus在mountain上,完全没有把前提当做lifestyle,一直再找削弱moutain使得长寿的选项,先入为主影响了判断;            
D, W是B地最受欢迎的旅游景点,无关;
E,W地人均休假活动消费比B地其他地区都高,无关; 如果需要做他因,还需要additional assumption,所以无关



作者: lululuna    时间: 2015-7-2 01:12
Prep2012-Pack1-CR-012        VCR005853        Easy
The faster a car is traveling, the less time the driver has to avoid a potential accident, and if a car does crash, higher speeds increase the risk of a fatality. Between 1995 and 2000, average highway speeds increased significantly in the United States, yet, over that time, there was a drop in the number of car-crash fatalities per highway mile driven by cars.

Which of the following, if true about the United States between 1995 and 2000, most helps to explain why the fatality rate decreased in spite of the increase in average highway speeds?

A.        The average number of passengers per car on highways increased.
B.        There were increases in both the proportion of people who wore seat belts and the proportion of cars that were equipped with airbags as safety devices.
C.        The increase in average highway speeds occurred as legal speed limits were raised on one highway after another.
D.        The average mileage driven on highways per car increased.
E.        In most locations on the highways, the density of vehicles on the highway did not decrease, although individual vehicles, on average, made their trips more quickly.

OA:B
P1: high speed increases risk
P2:    1995~2000, speed increases, but # fatality/mile (死人/mile) drop

explain the discrepancy

类型:他因型解释
A, passenger/car increases, 无关
B,    他因加强
C, 法律规定的速度上限, 与死亡了低无关
E, 车多了,与每迈死亡率无关

做错的原因,也是先入为主,太瞎了。
看到比例数字,立马想到了mile总数 变大了,看到miel的选项就选了;结果D项讲的是mile/car, 需要的是 total mile= mile/car * # car,其实不知道car的数量怎么变化的,D项并不能解释



作者: lululuna    时间: 2015-7-2 13:30
7/2
Find the main conclusion
source:
http://forum.chasedream.com/forum.php?mod=viewthread&tid=565843&extra=&authorid=579779&page=1

1) As one who has always believed that truth is our nation’s surest weapon in the propaganda war against our foes, I am distressed by reports of “disinformation” campaigns by American intelligence agents in Western Europe. In a disinformation campaign, untruths are disseminated through gullible local journalists in order to damage the interests of our enemies and protect our own. Those who defend this practice say that lying is necessary to counter Soviet disinformation campaigns aimed at damaging America’s political interests. These apologists contend that one must fight fire with fire. I would point out to the apologists that the fire department finds water more effective.

The author’s main point is that

(A) Although disinformation campaigns may be effective, they are unacceptable on ethical grounds
(B) America’s moral standing in the world depends on its adherence to the truth
(C) The temporary political gains produced by disinformation campaigns generally give way to long-term losses
(D) Soviet disinformation campaigns have done little to damage America’s standing in Europe
(E) Disinformation campaigns do not effectively serve the political interests of the United States

OA:E
main conclusion: I would point out to the apologists that the fire department finds water more effective.
fire= disinformation ,water =truth
A, disinformation campaigns may be effective(与结论相反)
B,moral standing,无关;
C,  无关,没提到longterm;
D, 未提及与苏联之间的胜负

2) Johnson is on firm ground when he asserts that the early editors of Dickinson’s poetry often distorted her intentions. Yet Johnson’s own, more faithful, text is still guilty of its own forms of distortion. To standardize Dickinson‘s often indecipherable handwritten punctuation by the use of the dash is to render permanent a casual mode of poetic phrasing that Dickinson surely never expected to see in print. It implies that Dickinson chose the dash as her typical mark of punctuation when, in fact, she apparently never made any definitive choice at all.
Which of the following best summarizes the author’s main point?

(A) Although Johnson is right in criticizing Dickinson’s early editors for their distortion of her work, his own text is guilty of equally serious distortions.
(B) Johnson’s use of the dash in his text of Dickinson’s poetry misleads readers about the poet’s intentions.
(C) Because Dickinson never expected her poetry to be published, virtually any attempt at editing it must run counter to her intentions.
(D) Although Johnson’s attempt to produce a more faithful text of Dickinson’s poetry is well meaning, his study of the material lacks sufficient thoroughness.
(E) Dickinson’s editors, including Johnson, have failed to deal adequately with the problem of deciphering Dickinson’s handwritten manuscripts.

OA:A
main conclusion:Yet Johnson’s own, more faithful, text is still guilty of its own forms of distortion


by sdcar2010:
Johnson is on firm ground when he asserts that the early editors of Dickinson’s poetry often distorted her intentions.Yet Johnson’s own, more faithful, text is still guilty of its own forms of distortion. To standardize Dickinson‘s often indecipherable handwritten punctuation by the use of the dash is to render permanent a casual mode of poetic phrasing that Dickinson surely never expected to see in print. It implies that Dickinson chose the dash as her typical mark of punctuation when, in fact, she apparently never made any definitive choice at all.

Opinion. Yet, conclusion. Premise.

3) Theater Critic: The play La Finestrina, now at Central Theater, was written in Italy in the eighteenth century. The director claims that this production is as similar to the original production as is possible in a modern theater. Although the actor who plays Harlequin the clown gives a performance very reminiscent of the twentieth-century American comedian Groucho Marx, Marx’s comic style was very much within the comic acting tradition that had begun in sixteenth-century Italy.

The considerations given best serve as part of an argument that

(A) Modern audiences would find it hard to tolerate certain characteristics of a historically accurate performance of an eighteenth-century play
(B) Groucho Marx once performed the part of the character Harlequin in La Finestrina
(C) In the United States the training of actors in the twentieth century is based on principles that do not differ radically from those that underlay the training of actors in eighteenth-century Italy
(D) The performance of the actor who plays Harlequin in La Finestrina does not serve as evidence against the director’s claim
(E) The director of La Finestrina must have advised the actor who plays Harlequin to model his performance on comic performances of Groucho Marx


OA D
Conclusion:The director claims that this production is as similar to the original production as is possible in a modern theater.
Concession : Although the actor who plays Harlequin the clown gives a performance very reminiscent of the twentieth-century American comedian Groucho Marx
Premise: Marx’s comic style was very much within the comic acting tradition that had begun in sixteenth-century Italy.

作者: lululuna    时间: 2015-7-2 20:52
7/3     
source:http://forum.chasedream.com/foru ... e=1&authorid=579779
Boldface part

3) More and more computer programs that provide solutions to mathematical problems in engineering are being produced, and it is thus increasingly unnecessary for practicing engineers to have a thorough understanding of fundamental mathematical principles. Consequently, in training engineers who will work in industry, less emphasis should be placed on mathematical principles, so that space in the engineering curriculum will be available for other important subjects.

The two portions in boldface play which of the following roles?

(A) The first is the second-premise that the argument includes; the second is the conclusion that could be drawn from this passage.
(B) The first is the fact that is necessary for this argument; the second is the conclusion that must be drawn from this passage.
(C) The first is the part of premise that the argument includes; the second is the inference that could be drawn from this passage

(D) The first is the part of evidence that supports this argument; the second is the inference that could be drawn from this passage.
(E) The first is the first conclusion in this argument; the second is the second conclusion in this argument.


OA:C
The two indicators "thus" and "consequently" point out the two possible conclusions in the argument.
(E) is an attractive answer choice. However, there is only one main conclusion in every argument. In this case the main conclusion is:
in training engineers who will work in industry, less emphasis should be placed on mathematical principles.
The second BF, which follows the phrase "so that," is simply an extension of what might happen if the main conclusion is reached. In logic, that is called inference


*不存在两个结论,如果有中间结论的话,不能说是conclusion,应该说是intermediate conclusion

6) The stable functioning of a society depends upon the relatively long-term stability of the goals of its citizens. This is clear from the fact that unless the majority of individuals have a predictable and enduring set of aspirations, it will be impossible for a legislature to craft laws that will augment the satisfaction of the citizenry, and it should be obvious that a society is stable only if its laws tend to increase the happiness of its citizens.

The claim that a society is stable only if its laws tend to increase the happiness of its citizens plays which one of the following roles in the argument?

(A) It is the conclusion of the argument.
(B) It helps to support the conclusion of the argument. (B)
(C) It is a claim that must be refuted if the conclusion is to be established.
(D) It is a consequence of the argument.
(E) It is used to illustrate the general principle that the argument presupposes.


red:conclusion;blue:premise




作者: lululuna    时间: 2015-7-5 23:08
7/5 assumption
7/28 review

source:http://forum.chasedream.com/foru ... e=1&authorid=579779
look for jumps in reasoning!!!


1. A recent report determined that although only 3 percent of drivers on Maryland highways equipped their vehicles with radar detectors, 33 percent of all vehicles ticketed for exceeding the speed limit were equipped with them. Clearly, drivers who equip their vehicles with radar detectors are more likely to exceed the speed limit regularly than are drivers who do not.
The conclusion drawn above depends on which of the following assumptions?
(A) Drivers who equip their vehicles with radar detectors are less likely to be ticketed for exceeding the speed limit than are drivers who do not.
(B) Drivers who are ticketed for exceeding the speed limit are more likely to exceed the speed limit regularly than are drivers who are not ticketed.
(C) The number of vehicles that were ticketed for exceeding the speed limit was greater than the number of vehicles that were equipped with radar detectors.
(D) Many of the vehicles that were ticketed for exceeding the speed limit were ticketed more than once in the time period covered by the report.
(E) Drivers on Maryland highways exceeded the speed limit more often than did drivers on other state highways not covered in the report.

OA:B
类型:断桥jump
P:33% V ticketed for excedding speed has rader
C:  V with rader are more likely to exceed speed
ticketed for excedding speed exceed speed

special tips: 这道题看了很久,也没选出来,脑子一看到数字就集中在了数字上,其实根据题型是assumption的,就应该从Jump这个点入手,不应该全部关注在数字上,
做题的时候还是容易抓小放大,看到题型要察觉到考察的point。assumption的题型,注意有没有未给出的假设才是关键,尤其是相近的差别只有一个单词的名词



2. A researcher discovered that people who have low levels of immune-system activity tend to score much lower on tests of mental health than do people with normal or high immune-system activity. The researcher concluded from this experiment that the immune system protects against mental illness as well as against physical disease.
The researcher's conclusion depends on which of the following assumptions?
(A) High immune-system activity protects against mental illness better than normal immune system activity does.
(B) Mental illness is similar to physical disease in its effects on body systems.
(C) People with high immune-system activity cannot develop mental illness.
(D) Mental illness does not cause people's immune system activity to decrease.
(E) Psychological treatment of mental illness is not as effective as is medical treatment.

OA:  D
P: low immue sys activity correlate with low tests of mental health
C: immue sys protect against mental health as well as physical diseases

典型的A和B同时发生,a是b 的原因
assumption应该是 b不是a 的原因
正确选项恰恰是讲了b不是a的原因,使得假设更完整。
错的原因:看到physical diseases 之后就想到了jumps,此题确实存在jumps,但是keypoint并不是断桥;
                 根据前提和结论,as well as, physical diseases 只是顺带提及;
                 再者,b项看似消除了这个jump其实不是,因为稳重没说到mental illness对body sys的effect是怎样的,
                 那么b项需要additional assumption,就是mental health对body sys有什么effect,并不能是原文的assumption更完整



3. In Kravonia, the average salary for jobs requiring a college degree has always been higher than the average salary for jobs that do not require a degree. Current enrollments in Kravonia's colleges indicate that over the next four years the percentage of the Kravonian workforce with college degrees will increase dramatically.  Therefore, the average salary for all workers in Kravonia is likely to increase over the next four years.
Which of the following is an assumption on which the argument depends?
A. Kravonians with more than one college degree earn more, on average, than do Kravonians with only one college degree.
B. The percentage of Kravonians who attend college in order to earn higher salaries is higher now than it was several years ago.
C. The higher average salary for jobs requiring a college degree is not due largely to a scarcity among the Kravonian workforce of people with a college degree.
D. The average salary in Kravonia for jobs that do not require a college degree will not increase over the next four years.
E. Few members of the Kravonian workforce earned their degrees in other countries.

OA: C
P1: now, salary for degree holder is higher
P2: current enrollment indicates % workforce with degree increase in the next four yrs
C: avg salary will increase in next four yrs

two flaws:
1) now/past future
2) jump : enrollmet in college workforce in mkt( 这题有个weaken,就是选学生enroll 了不见得会毕业)

9. The average hourly wage of television assemblers in Vernland has long been significantly lower than that in neighboring Borodia. Since Borodia dropped all tariffs on Vernlandian televisions three years ago, the number of televisions sold annually in Borodia has not changed. However, recent statistics show a drop in the number of television assemblers in Borodia. Therefore, updated trade statistics will probably indicate that the number of televisions Borodia imports annually from Vernland has increased.
Which of the following is an assumption on which the argument depends?
(A) The number of television assemblers in Vernland has increased by at least as much as the number of television assemblers in Borodia has decreased.
(B) Televisions assembled in Vernland have features that televisions assembled in Borodia do not have.
(C) The average number of hours it takes a Borodian television assembler to assemble a television has not decreased significantly during the past three years.
(D) The number of televisions assembled annually in Vernland has increased signifi cantly during the past three years.
(E) The difference between the hourly wage of television assemblers in Vernland and the hourly wage of television assemblers in Borodia is likely to decrease in the next few years.

OA:C
P: # TV assembler decrease, sale remains
C: # TV imports increase

demand = supply
demand keep , supply decrease, so import increase
supply= TV assembler # * TV/ assembler/ hour * # hour





















作者: lululuna    时间: 2015-7-6 19:39
7/6
7/29review

Q17. p1-cr 36. (32124-!-item-!-188;#058&006025)
Outsourcing is the practice of obtaining from an independent supplier a product or service that a company has previouslyprovided for itself.  Vernon, Inc., asmall manufacturing company that has in recent years experienced a decline inits profits, plans to boost its profits by outsourcing those parts of its business that independent suppliers can provide at lower cost than Vernon can itself.

P: outsource business that independent suppliers can provide at lower cost than Vernon
C: to increase profit

Which of the following, if true, most stronglysupports the prediction that Vernon's plan will achieve its goal?
(A) Among the parts of its business that Vernondoes not plan to outsource are some that require standards of accuracy too highfor most independent suppliers to provide at lower cost than Vernon can.
(B) Vernon itself acts as an independent supplierof specialized hardware items to certain manufacturers that formerly made those items themselves.
(C) Relatively few manufacturers that start as independent suppliers have been able to expand their business and become direct competitors of the companies they once supplied.
(D) Vernon plans to select the independent suppliers it will use on the basis of submitted bids.
(E) Attending to certain tasks that Vernon performs relatively inefficiently has taken up much of the time and effort of top managers whose time would have been better spent attending to Vernon's core business.

OA:E
A/B无关
其实我是没法儿判断出选E的,C/D的程度并不比E差,
E并不能证明take up time的那些low efficiently的工作是V被外包的那部分工作
can provide at lower cost than Vernon can itself =?? certain tasks that Vernon performs relatively inefficiently


Q21.  p1-cr 37.  (32356-!-item-!-188;#058&006101)
Capuchin monkeys in Venezuela often rub a certaintype of millipede into their fur. Secretions of these millipedes have been shown to contain two chemicalsthat are potent mosquito repellents, and mosquitoes carry parasites that debilitate the capuchins.  The rubbing behavior is rare except during the rainy season, when mosquito populations are at their peak.  Therefore, the monkeysprobably rub the millipedes into their fur because doing so helps protect them against mosquitoes.

Which of the following would it be most useful to determine in order to evaluate the argument?
(A) Whether the two chemicals provide anyprotection for the millipedes against their own predators
(B) Whether the type of millipede used by the capuchin monkeys in Venezuela is found in other parts of the world
(C) Whether animals other than capuchins rub insects of any kind into their fur
(D) Whether the only time the millipedes are readily available to thecapuchins is during the rainy season
(E) Whether secretions of any other insectsaccessible to the capuchins contain chemicals that repel mosquitoes

OA:  d

C rub M=> M contain chemical=> chemical is mosquito repellent
Mosquito carry parasites debilitate C

P: rub is rare except in rainy season (mosquito is rich )
C: monkey rub M to protect against mosquito
=> 因: mosquito  in rainy season
     果:    monkey rub M to  repell mosquito
==> OA: M is only available in rainy season =>  weaken the conclusion
              M can be caugth any time in the year, but monkey catch only in rainy season => enhance the conclusion

Q29.     p1-cr    40.        (32869-!-item-!-188;#058&006461)
Columnist: People should avoid using a certain artificial fat that has been touted as a resource for those whose medical advisers have advised them to reduce their fat intake.  Although the artificial fat, which can be used in place of fat in food preparation, has none of the negative health effects of fat, it does have a serious drawback:  it absorbs certain essential vitamins,thereby preventing them from being used by the body.

In evaluating the columnist's position, it wouldbe most useful to determine which of the following?
(A) Whether increasing one's intake of the vitamins can compensate for the effects of the artificial fat
(B) Whether the vitamins that the artificial fat absorbs are present in foods that contain the fat
(C) Whether having an extremely low fat intake for an extended period can endanger the health
(D) Whether there are any foods that cannot be prepared using the artificial fat as a substitute for other fats
(E) Whether people are generally able to detect differences in taste between foods prepared

oa:a
P: AF  absorb V which could have been used by body
C: avoid AF for those whose are advised to reduce fat intake

作者: lululuna    时间: 2015-7-8 21:42
7/8
7/29review

PREP7 第七套

Q5.       p1-cr    45.        (33847-!-item-!-188;#058&007123)
A major impediment to wide acceptance of electricvehicles even on the part of people who use their cars almost exclusively for commuting is the inability to use electric vehicles for occasional extended trips.  In an attempt to make purchasing electric vehicles more attractive to commuters, one electric vehicle produceris planning to offer customers three days free rental of a conventional car forevery 1,000 miles that they drive their electric vehicle.
Which of the following, if true, most threatensthe plan's prospects for success?
(A) Many electric vehicles that are used for commercial purposes are not needed for extended trips.
(B) Because a majority of commuters drive at least 100 miles a week, the cost to the producer of making good the offer would add considerably to the already high price of electric vehicles.
(C) The relatively long time it takes to rechargethe battery of an electric vehicle can easily be fitted into the regularpatterns of car use characteristic of commuters.
(D) Although electric vehicles are essentiallyemission-free in actual use, generating the electricity necessary for chargingan electric vehicle's battery can burden the environment.
(E) Some family vehicles are used primarily notfor commuting but for making short local trips, such as to do errands.

OA:B

p: plan to offer 3 day free rentel of  conventional car for every  1000 miles they use electric vehicle
C: to make purchasing electric vehicle attractive


作者: lululuna    时间: 2015-7-9 00:00
7/8 VS 6/30两道很相似的题

Prep2-24(28903-!-item-!-188;#058&004064)Although exposure to asbestos is  the primary cause of mesothelioma, a slow-developing cancer, researchers  believe that infection by the SV40 virus is a contributing cause, since in  the United States 60 percent of tissue samples from mesotheliomas, but none  from healthy tissue, contain SV40. SV40 is a monkey virus; however, in 1960  some polio vaccine was contaminated with the virus. Researchers hypothesize  that this vaccine was the source of the virus found in mesotheliomas decades  later.
  Which of the following, if true, most  strongly supports the researchers'  hypothesis?
  
  (A) SV40 is widely used as a research tool in cancer laboratories.
  (B) Changes in the technique of manufacturing the vaccine now prevent  contamination with SV40.
  (C) Recently discovered samples of the vaccine  dating from 1960 still show traces of the virus.
  (D) In a small percentage of cases of mesothelioma, there is no history of  exposure to asbestos.
  (E) In Finland, where the polio vaccine was never contaminated, samples from  mesotheliomas do not contain SV40.
  
  
P: vaccine  was contaminated with SV40 in 1960S
  C: vaccine was the source of the SV40 found in M decades later
  
  E, no V contamined, no SV40 in M
   否命题
  
  
Q16  p1-cr46. (33987-!-item-!188;#058&007200) Archaeologists in Michigan have excavated a  Native American camp near Dumaw Creek.   Radiocarbon dating of animal bones found at the site indicates that  the camp dates from some time between 1605 and 1755.  However, the camp probably dates to no  later than 1630, since no European trade goods were found at the site, and  European traders were active in the region from the 1620's onward.
  Which  of the following, if true, most strengthens the argument?
   
  (A) Due to trade among Native Americans, some European trade goods would  have reached the area before the European traders themselves did. Weaken
  (B) At all camps in the region that  have been reliably dated to the late 1620's, remains of European trade goods  have been found.   
  (C) The first European trade goods to reach the area would have been  considered especially valuable and preserved as much as possible from loss or  destruction.
  (D) The first European traders in the area followed soon after the first  European explorers
  (E) The site is that of a temporary camp that would have been used  seasonally for a few years and then abandoned.
P: no E trade goods found, but E  active in 1620s onward
  C: camp dates to no later 1630(<=1920)
  
  B, for all camp date to late 1620s,E good found
  
  
  因:早在16世纪20年代E就开始活动了,但却没有E被找到
  果: camp不晚于1630年,或者更早
  
  B,所有camp被确认为在16世纪20年代末,这些E的保留物已经能被找到了
  
  B项的年代范围比题目里的范围更窄


作者: lululuna    时间: 2015-7-9 15:46
7/9

Q3.  p1-cr 1.  (24179-!-item-!-188;#058&000705)
Although fullerenes--spherical molecules made entirely of carbon--were first found in the laboratory, they have since been found in nature, formed in fissures of the rare mineral shungite.  Since laboratory synthesis of fullerenes requires distinctive conditions of temperature and pressure, this discovery should give geologists a test case for evaluating hypotheses about the state of the Earth's crust at the time these naturally occurring fullerenes were formed.

P:laboratory synthesis of fullerenes requires distinctive temperature and pressure
C: give geologists a test case for evaluating hypotheses aboutthe state of the Earth's crust at the time these natural fullerenes formed

Whichof the following, if true, most seriously undermines the argument?
(A)Confirming that the shungite genuinely contained fullerenes took careful experimentation.
(B)Some fullerenes have also been found on the remains of a small meteorite that collided with a spacecraft.
(C)The mineral shungite itself contains large amounts of carbon, from which the fullerenes apparently formed.
(D) The naturally occurring fullerenes are arranged in a previously unknown crystalline structure.
(E)Shungite itself is formed only under distinctive conditions.


OA:  D

Q4.  p1-cr  2.    (24227-!-item-!-188;#058&000711)

Economist:  On average, the emergency treatment for anelderly person for injuries resulting from a fall costs $11,000.  A new therapeutic program can significantly reduce an elderly person's chances of falling. Though obviously desirable for many reasons, this treatment program willcost $12,500 and thus cannot be justified.

Whichof the following, if true, most seriously undermines the conclusion of theargument?

(A)Among elderly people who had followed the program for only a few months, the number of serious falls reported was higher than it was for people who had followed the program for its recommended minimum length of one year.
(B)Falls resulting in serious injuries are less common among elderly people living in nursing homes than they are among elderlypeople who live alone at home.
(C)A frequent result of injuries sustained in falls is long-term pain, medication for which is not counted among the average per-person costs of emergency treatment for elderly people's injuries from such falls.
(D)The new therapeutic program focuses on therapies other than medication, since overmedication can cause disorientation and hence increase the likelihood that an elderly person will have a serious fall.
(E)A significant portion of the cost of the new therapeutic program is represented by regular visits by health care professionals, the costs of which tend to increase more rapidly than do those of other elements of the program.

OA:  c

P: cost (12500>11000)
C: plan (reduce chances of falling of the elder ) can’t be justified,
WEAKEN

迷惑项D: increase the likehood of falling , but the premise for rejecting the plan is the higher cost, it has nothing to do with the likehood of falling between two, actually the difference in likehood of falling is already stated in the stimuli in the Question.
*special attention: 重复题目bg类型题,对我挺有迷惑性的,take care!!
作者: lululuna    时间: 2015-7-10 00:13
7/9
7/29 review

Q40.    p1-cr    12.        (25359-!-item-!-188;#058&001894)
Finding of a survey of Systems magazinesubscribers:  Thirty percent of all merchandise orders placed by subscribers in response to advertisements in the magazine last year were placed by subscribers under age thirty-five.
30% orders (by subscribers in response to ads)are placed by subscriber <35
因为广告而买的订阅者当中,30%是小于35岁的订阅者
Finding of a survey of advertisers in Systems magazine:  Most of the merchandise orders placed in response to advertisements in Systems last year were placed by people under age thirty-five.
Most orders respond to ads  are placed by people<35
因为广告而产生的订单中,大部分来自于小于35岁的订阅者

For both of the findings to be accurate, which ofthe following must be true?
(A) More subscribers to Systems who have never ordered merchandise in response to advertisements in the magazine are age thirty-five or over than are under age thirty-five.
(B) Among subscribers to Systems, the proportion who are under age thirty-five was considerably lower last year than it is now.
(C) Most merchandise orders placed in response to a dvertisements in Systems last year were placed by Systems subscribers over agethirty-five.
(D) Last year, the average dollar amount ofmerchandise orders placed was less for subscribers under age thirty-five thanfor those age thirty-five or over.
(E) Last year many people who placed orders for merchandise in response to advertisements in Systems were not subscribers to the magazine.


作者: lululuna    时间: 2015-7-12 16:14
7/12
7/29review

Q19. p2-rc17. (26864-!-item-!-188;#058&003368)

At present the Hollywood Restaurant has only standard-height tables.  However, manycustomers come to watch the celebrities who frequent the Hollywood, and theywould prefer tall tables with stools because such seating would afford a betterview of the celebrities.  Moreover,diners seated on stools typically do not stay as long as diners seated at standard-heighttables.  Therefore, if the Hollywoodreplaced some of its seating with high tables and stools, its profits wouldincrease.

P: diners seated on stools < diners seated at standard-height tables @time
C: replace  seating with high tables and stools, its profits would increase.

The argument is vulnerable to criticism on the grounds that it givesreason to believe that it is likely that
(A) some celebrities come to the Hollywood to beseen, and so might choose to sit at the tall tables if they were available
(B) the price of meals ordered by celebritiesdining at the Hollywood compensates for the longer time, if any, they spendlingering over their meals
(C) a customer of the Hollywood who would chooseto sit at a tall table would be an exception to the generalization aboutlingering
(D) a restaurant's customers who spend less timeat their meals typically order less expensive meals than those who remain attheir meals longer
(E) with enough tall tables to accommodate all theHollywood's customers interested in such seating, there would be no view exceptof other tall tables


SDCAR2010: 3#
SOURCE:http://forum.chasedream.com/thread-565901-1-1.html
The reason to eliminate D) is that this question is a must be true type of question, meaning you cannot introduce NEW information. D) introduces NEW information while C) only points out the self-controversy stated in the stimulus.

If you think about it, how can you deduce D) by reading the stimulus ONLY? The prompt directs you to find something which "gives reason to believe that it is likely" according to the passage.

The following is the complete analysis.

First of all, this is similar to a paradox question and the question stem asks you to find the criticism which points out the deficiency in the argument.  So let's analyze the argument.

Premises:
1) Customers come to Hollywood Restaurant to watch the celebrities so customrs would prefer tall tables to get a better view.
2) Diners seated on stools typically stay a shorter time than diners on regular seats.

Conclusion:
If the Hollywood replaced some of its seating with high tables and stools, its profits would increase.


Basically, the argument says that stools would attract more customers and customers sitting on stools turn over quickly.  Therefore, profits would be up.  Wait a minute.  Based on premise 1, if the customers are attracted to the restaraunt because they want to see celebrities, shouldn't they stay LONGER than normal customers? If so, it runs contrary to premise 2 which describes a general trend in customer's lingering behavior. The customer attracted might sit on the stools for a LONNNNNNNNNNNNNNG time without spending much on food. No turnover, no money!

C points out this paradox and C is the correct answer.

作者: lululuna    时间: 2015-7-13 00:35
类别:化石/遗迹 没留下 不等于 没有出现过

Prep2012-Pack1-CR-035       VCR001345 Medium
Perkins: According to an article I read, the woolly mammoth‘s extinction in North Americacoincided with a migration of humansonto the continent12,000 years ago, and stone spearheads from this period indicatethat these people were hunters. But the author's contention that being hunted by humans contributed to thewoolly mammoth‘s extinction is surelywrong since, as paleontologists know, no spearheads have ever been found among the many mammothbones that have been unearthed.

Which of the following, if true, provides the strongest reason for discounting the evidencePerkins cites inarguing againstthe contention that being hunted by humans contributed to the North American extinction of woollymammoths?
A.         At sites where mammoth bones dating from 12,000years ago have been unearthed, bones of other mammalshave rarely been found.
B.          The stone from whichstone spearheads were made is unlikely to have disintegrated over the course of 12,000 years.  Stone stilled integrated but notfound, strengthen
C.         Conditions in North America 12,000years ago were such thathumans could not have survivedthere on a diet that did not includesubstantial amounts of meat.  
D.         Cave paintings in North Americathat date from 12,000 years ago depict woolly mammothsas well as a varietyof other animals,including deer and buffalo.
E.          Because of the great effort that would have been  requiredto produce each stone spearhead, hunters would not have been to leave thembehind.

Mine:

P: no spearheads have ever been found among the many mammoth bones

C: hunter is not responsible for the wolly extinction


Weaken=> spearheads has been taken away? hunters use spear to hunt animal? 下面引用部分的彩色部分

Reasoning
What would suggest thathunting by humans may have contributed to the mammoths' extinction, even thoughno spearheads have been found among mammoth bones? Perkins argues that since no spearheadshave been found among mammoth remains, human hunting must not have contributedto the mammoths’ extinction. Perkins's argument assumes that humans did nothunt mammoths mainly with weapons other than spears; that spearheads would beleft among remains of any mammoths killed with spears; that paleontologistshave found a fair number of mammoth remains; and that any spearheads left amongthe remains would likely stay with them and be discovered. Any statement thatcalls any of these assumptions into question would provide a reason to discountthe evidence Perkins offers.

A. If humans did not hunt mammoths, then mammothbones probably would not be found with other mammal bones around prehistoricsettlements where humans ate other mammals. Therefore, this would be compatiblewith Perkins’ argument.
B. This strengthens Perkins’ argument by out apossible alternative explanation of why paleontologists have found nospearheads in mammoth remains.
C. The prehistoric humans could have eaten meatother than mammoths, and this does not explain how they could have huntedmammoths without paleontologists finding spearheads in mammoth remains.
D. Perkins already acknowledges that humanscoexisted with mammoths in North America, and this does not explain how humanscould have hunted mammoths without paleontologists finding spearheads inmammoth remains.
E.Correct.This explains how, despite the evidence Perkins offers, humans may well havehunted mammoths without leaving any spearheads for paleontologists to discoverin mammoth remains.
Thecorrect answer is E.

Which of the following most logically completes the argument?

The last members of a now-extinct species of a European wild deer called the giant deer lived in Ireland about 16,000 years ago.  Prehistoric cave paintings in France depict this animal as having a large hump on its back. Fossils of this animal, however, do not show any hump.  Nevertheless, there is no reason to conclude that the cave paintings are therefore inaccurate in this regard, since __________.

(A) some prehistoric cave paintings in France also depict other animals as having a hump
(B) fossils of the giant deer are much more common in Ireland than in France
(C) animal humps are composed of fatty tissue, which does not fossilize
(D) the cave paintings of the giant deer were painted well before 16,000 years ago
(E) only one currently existing species of deer has any anatomical feature that even remotely resembles a hump

P:no hump in fossil; hump in pic
C:   the pic is correct
OA: C



7/20
化石、遗迹有留存 本来就存在
同源题
GWD30-Q38:
Theancient Nubians inhabited an area in which typhus occurred, yet surprisinglyfew of their skeletons show the usual evidence of this disease.  The skeletons do show deposits oftetracycline, an antibiotic produced by a bacterium common in Nubian soil.  This bacterium can flourish on the dried grainused for making two staples of the Nubian diet, beer and bread.  Thus, tetracycline in their food probablyexplains the low incidence of typhus among ancient Nubians.
P: Nubian skeletons no evidence of typhus(disease) but deposits of tetracycline(antibiotic)
C: food ( tetracycline fluorished on dried grain the staple of Nubian) explains the low incidence

gap:  deposits of bactria ≠ bacteria in food=》 possibly from other resoures

Which of the following is an assumption on which the argumentrelies?
A.    The tetracycline deposits did not form after the bodies were buried.
B.     The diseases other than typhus to which the ancient Nubians wereexposed would not be affected by tetracycline.
C.    Typhus is generally fatal.
D.    Nubian grain became contaminated with tetracycline-producingbacteria prior to being harvested.
E.     Bread and beer were the only foods eaten by the ancient Nubianswhich could have contained tetracycline.
oa: A

GWD18-Q30
The ancient Nubians inhabited anarea in which typhus occurs, yet surprisingly few of their skeletons show theusual evidence of this disease.  Theskeletons do show deposits of tetracycline, an antibiotic produced by abacterium common in Nubian soil.  Thisbacterium can flourish on the dried grain used for making two staples of theNubian diet, beer and bread.  Thus,tetracycline in their food probably explains the low incidence of typhus amongancient Nubians.
Which of the following is an assumption on whichthe argument relies?
A.   Infectiousdiseases other than typhus to which the ancient Nubians were exposed areunaffected by tetracycline.
B.    Tetracycline is not rendered ineffective as anantibiotic by exposure to the processes involved in making bread and beer.
C.   Typhuscannot be transmitted by ingesting bread or beer contaminated with theinfectious agents of this disease.
D.   Breadand beer were the only items in the diet of the ancient Nubians which couldhave contained tetracycline.
E.    Typhusis generally fatal.

OA: B
P: Nubian skeletons no evidence of typhus(disease) but deposits of tetracycline(antibiotic)
C: food ( tetracycline fluorished on dried grain for the staple of Nubian) explains the low incidence

gap:  deposits of bactria ≠ bacteria in food=》 possibly from other resoures


有某地的遗迹 不等于 就是某地的人/某地人的产物、

Prep2012-Pack1-CR-044       VCR005074     Medium
The Nile Delta of Egypt was invaded and ruled from 1650 to 1550 B.C. by a peoplecalled the Hyksos.Their origin is uncertain, but archaeologists hypothesize that they were Canaanites. In supportof this hypothesis, the archaeologists point out that excavations of Avaris,the Hyksos capitalin Egypt, have uncovered large numbersof artifacts virtuallyidentical toartifacts producedin Ashkelon, a major city of Canaan atthe time of the Hyksosinvasion.

In order to evaluate the forceof the archaeologists’ evidence,it would be most useful to determinewhich of the following?
A.         Whether there were some artifactsfound at Avaris thatwere unlike those produced in Ashkelonbut that date to before 1700 B.C.
B.          Whether the Hyksos ruled any other part of Egypt besidesthe Delta in the period from 1650 to 1550 B.C.
C.         Whether Avaris was the nearest Hyksoscity in Egypt to Canaan
D.         Whether Ashkelon after 1550 B.C. continuedto produce artifacts similar to those found at Avaris
E.          Whether many of the artifactsfound at Avaris that are similarto artifacts producedin Ashkelon date to well before the Hyksos invasion
OA: E

GWD 31-10  与上一题同题源The Hyksos invaded the Nile Delta of Egypt and ruled it from 1650 B.C. Their origin is uncertain, but archaeologists hypothesize that they were Canaanites. In support of this hypothesis, the archaeologists point out that excavations of Avans, the Hyksos capital in Egypt, have uncovered large numbers of artifacts virtually Identical to artifacts produced in Ashkelon, and Ashkelon was a major city of Canaan at the time of the Hyksos’ invasion.

In order to evaluate the force of the archaeologists’ evidence, it would useful to determine which of the following?
A: Whether artifacts from Ashkelon were widely traded to non-Canaanite cities?
B: Whether significant numbers of artifacts that do not resemble artifacts produced in Ashkelon have been found at Avans?
C: Whether Avans was the nearest Hyksos city in Egypt to Canaan?
D: Whether Ashkelon after 1550 B.C. continued to produce artifacts similar to those found at Avans?
E: whether any artifacts produced by the Hyksos after 1550 B.C .have been found in Egypt

OA: A


GWD-23-Q32
Excavations of the Roman city of Sepphoris have uncovered numerous detailed mosaics depicting several readily identifiable animal species:  a hare, a partridge, and various Mediterranean fish.  Oddly, most of the species represented did not live in the Sepphoris region when these mosaics were created.  Since identical motifs appear in mosaics found in other Roman cities, however, the mosaics of Sepphoris were very likely created by traveling artisans from some other part of the Roman Empire.
Which of the following is an assumption on which the argument depends?


OA: E

作者: lululuna    时间: 2015-7-15 14:37
7/15
ALL THINGS EQUAL型题目

PREP08-1-Q8
74.   (29534-!-item-!-188;#058&004138)
Ina certain wildlife park, park rangers are able to track the movements of manyrhinoceroses because those animals wear radio collars.  When, as often happens, a collar slips off,it is put back on.  Putting a collar on arhinoceros involves immobilizing the animal by shooting it with a tranquilizerdart. Female rhinoceroses that have been frequently recollared havesignificantly lower fertility rates than uncollared females.  Probably, therefore, some substance in thetranquilizer inhibits fertility.
Inevaluating the argument, it would be most useful to determine which of thefollowing?

A.Whether there are more collared female rhinoceroses than uncollared femalerhinoceroses in the park
B.How the tranquilizer that is used forimmobilizing rhinoceroses differs, if at all, from tranquilizers used inworking with other large mammals
C.How often park rangers need to use tranquilizer darts to immobilizerhinoceroses for reasons other than attaching radio collars
D.Whether male rhinoceroses in the wildlife park lose their collars any moreoften than the park’s female rhinoceroses do
E.Whether radio collars are the only practical means that park rangers have fortracking the movements of rhinoceroses in the park

OA: C

BG: T is required when recollaring
P: frequent recollared female < unrecollared female @  fertality rates
C:  sth in T inhibits fertility

做这题的时候是用无关选项排除的
看C项,如果出了recollar之外,还常用T来使犀牛镇定下来,那么如果这个频率非常高,远高于recollar的计量,那么recollar的计量可以忽略不计=》削弱STH in T降低生育率;
             若果出了recollar之外,几乎不用T,那么两个group( recollared Femalle和 unrecollared female)之间很大的一个区别就是T的剂量=> 加强了前提

只提因素A,假设其他“all things equal”型错误,参考AWA

PREP08-01-Q9
75.   (33198-!-item-!-188;#058&007254)
Inthe year following an eight-cent increase in the federal tax on a pack ofcigarettes, sales of cigarettes fell ten percent.  In contrast, in the year prior to the taxincrease, sales had fallen one percent. The volume of cigarette sales is therefore strongly related to theafter-tax price of a pack of cigarettes.
Whichof the following, if true, would most strengthen the argument above?

A.During the second year after the tax increase, cigarette sales increased by asignificant amount.
B.The information available to consumers on the health risks of smoking remainedlargely unchanged in the period before and after the tax increase.
C.Most consumers were unaware that the tax on cigarettes was going to increase.
D.During the year following the cigarette tax increase, many consumers had lessincome, in inflation-adjusted dollars, than they had had in the previous year.
E.During the year after the tax increase, there was a greater variety ofcigarettes on the market than there had been during the previous year.

OA: B

P: yrs after increases in tax rate on cig, sale of cig decreases
C: increased tax causes the fall in cig sale

只提tax rate的变化,对其他因素没有描述,假设all else things equal是这个argument本身的缺陷
正确选项的加强方式,是通过讲 其他因素维持不变,加强前提,进而加强假设


作者: lululuna    时间: 2015-7-15 15:40
同源题
downtown shopping center 和 SAVE discount store

PREP08_01_Q17
77.   (30754-!-item-!-188;#058&005525)
Whichof the following most logically completes the argument below?
Twentypercent of the stores in Morganville'sdowntown shopping district will fail within five years because they willbe competing directly with the SaveMart discount department store newly opened in East Morganville.  The downtown shopping district has lostbusiness at this rate before and has always completely rebounded.  Confidence that itwill rebound again from the losses it is now about to suffer is ill founded,however, because __________.
A. the stores likely to be put out ofbusiness by direct competition from SaveMart are the downtown shoppingdistrict's anchor stores, on whose ability to draw shoppers many of the otherdowntown stores depend
B.the bus line that has long connected the downtown area of Morganville with EastMorganville has a tradition of carrying shoppers who reside in East Morganvilleinto downtown Morganville to shop
C.when the downtown shopping district has rebounded before, the business premises of a failed business were typically taken over by a business of the same kindas had been there before
D.SaveMart's business plan for the East Morganville store is based on earning lowprofits, if any, during the first five years of the store's existence
E.it is conceivable that the downtown shopping district could shrinksubstantially without collapsing altogether

BG: stores in M's downtown competing directly with the SaveMart discount department store newly opened in East M=> 20% fail in downtown M
P:    The downtown shopping district has lost business at this rate before and has always completely rebounded
C:    will not rebound this time

ALL THINGS EQUAL错误
过去不等于现在=》指出两个time period的不同之处


OG13_70
Community activist: If Morganville wants to keep its central shopping district healthy, it should prevent the opening of a huge SaveAII discount department store on the outskirts of Morganville. Records from other small towns show that whenever SaveAII has opened a store outside the central shopping district of a small town, within five years the town has experienced the bankruptcies of more than a quarter of the stores in the shopping district.

The answer to which of the following would be most useful for evaluatingthe community activist's reasoning?
(A) Have community activists in other towns successfully campaigned against the opening of a SaveAII store on the outskirts of their towns?
(B) Do a large percentage of the residents of Morganville currently do almost all of their shopping at stores in Morganville?
(C) In towns with healthy central shopping districts, what proportion of the stores inthose districts suffer bankruptcy during a typical five-year period?
(D) What proportion of the employees at the SaveAII store oh the outskirts of Morganville will be drawn from Morganville?
(E) Do newly opened SaveAII stores ever lose money duringtheir first five years of operation?

OA: C
P:previously, SAVEALL opened, other town face >1/4 stores bankruptcies
C : prevent open SAVEALL at M to keep shopping district healthy

断桥,connect % with the definition

OG13_107
Although the discount stores in Goreville's central shopping districtare expected to close within five years as a result of competition from a SpendLess discount department store that just opened, those locations will not stay vacant for long. In the five years since the opening of Colson's, a nondiscount department store, a new store has opened at the location of every store in the shopping district that closed because it could not compete with Colson's.

Which of the following, if true, most seriously weakens the argument?
(A) Many customers of Colson's are expected to do less shopping there than they did before the SpendLess store opened.
(B) Increasingly, the stores that have opened in the central shopping district since Colson's opened have been discount stores.
(C) At present, the central shopping district has as many stores operating in it as it ever had.
(D) Over the course of the next five years, it is expected that Goreville's population will grow at a faster rate than it has for the past several decades.
(E) Many stores in the central shopping district sell types of merchandise that are not available at either SpendLess or Colson's.

OA: B

P: C(nondiscount) opened, many stoere closed locations X remian empty, new stores ioen at that location;
C: although SPENDLESS( discount store), G‘s central shopping closed may stores ,but won't stay empty for a long time.
What happend to C, is different from Spendless, point out the differrence between the two is the way to go.

false analogy




作者: lululuna    时间: 2015-7-15 20:32
7/15

Prep2012-Pack1-CR-039       VCR003744     Medium
Which of the followingmost logically completes the passage?
Each species of moth has an optimalbody temperature for effective flight, and when air temperatures fall much  below that temperature, the moths typically have to remain inactiveon vegetation for extendedperiods, leaving  them highly vulnerable to predators. Ingeneral, larger moths can fly fasterthan smaller ones and hence have a better chanceof evading flying predators, but they also have higheroptimal body temperatures, which explainswhy_____
                  .
A.   large moths are generally able to maneuver better in flight than smallermoths
B.   large moths are proportionally much more commonin warm climatesthan in cool climates
C.   small moths are more likely than large moths to be effectively camouflaged whileon vegetation
D.   large moths typically have wingsthat are larger inproportion to their body size than smallermoths do
E.   most predators of moths preynot only on severaldifferent species of moth butalso on various species of other insects

P:  air temperature < optimal body temperature for flight
C: moths are vulnerable to predator

because higher body temperature, large moths are more vulnerable to predator in cool climates than in warm climteas
                                         =>  large M are more likely to die in cool climates than in warm climates
                                         =>  large M are more common in warm climates than in cool climates( because of the mortality rates difference)

这题逻辑挺简单的,但问题是坑特别大,But后半句跟前面和small moth的比较是无关的,
but之后的句子自成体系,给出了(higher body temperature) 和果____(mortality diffrence in different climate areas)


作者: lululuna    时间: 2015-7-19 00:33
7/17

19. GWD31-19
Sonya: The government of Copeland israising the cigarette tax. Copeland’s cigarette priceswill still be reasonably low, so cigarette consumption will probably not beaffected much. Consequently, government revenue from the tax willincrease.
Raoul: True, smoking is unlikely todecrease, because Copeland’s cigarette prices will still not be high. They will, however, no longer be the lowest in the region, sowe might begin to see substantial illegal sales of smuggled cigarettes inCopeland.
Raoul responds to Sonya’s argument by doingwhich of the following?
What mentions in R but not in  S is the prc  will decrease as a result of relative higher prc than prcs of other cities in the region,
and R used the increased prc to refute S
A.    Questioningthe support for Sonya’s conclusion by distinguishingcarefully between no change and no decrease=wrong draft
B.     calling Sonya’sconclusion into question by pointing to a possible effect of a certain change.
C.    Arguingthat Sonya’s conclusion would be better supported if Sonya could cite aprecedent for what she predicts will happen.  refute S
D.    showingthat a cause that Sonya claims will be producing a certain effect is not theonly cause that could produce that effect
E.     pointingout that a certain initiative is not bold enough to have the predicts it willhave

  错选了e, 应该把虚词代回原文 predict = revenue will increase
  R refute S 不是说某个措施not bold enough,而是该措施 不能


作者: lululuna    时间: 2015-7-19 01:12
7/20  一道黑脸题的众多变体(难)
同源题
GWD30-Q26:
In countries where automobile insurance includes compensation for whiplash injuries sustained in automobile accidents, reports of having suffered suchinjuries are twice as frequent as they are in countries where whiplash is notcovered.  Some commentators have argued, correctly, that since there is presently no objectivetest for whiplash, spurious reports of whiplash injuries cannot be readily identified.  These commentators are, however, wrong to draw the further conclusion that inthe countries with the higher rates of reported whiplash injuries, half of thereported cases are spurious:  clearly, incountries where automobile insurance does not include compensation forwhiplash, people often have little incentive to report whiplash injuries thatthey actually have suffered.

Inthe argument given, the two boldfaced portions play which of the followingroles?
A.    The first is a finding whose accuracy is evaluated in the argument; the second isan intermediate conclusion drawn to support the judgment reached by theargument on the accuracy of that finding.
B.     The first is a finding whose accuracy is evaluated in the argument; the second isevidence that has been used to challenge the accuracy of that finding.
C.    The first is a finding whose implications are at issue in the argument; the secondis an intermediate conclusion that has been used to support a conclusion thatthe argument criticizes.
D.    The first is a claim that the argument disputes; thesecond is a narrower claim that the argument accepts.
E.     The first is a claim that has been used to support aconclusion that the argument accepts; the second is that conclusion.   
OA: C

GWD30-Q28:
In countries where automobile insurance includes compensation for whiplash injuries sustained in automobile accidents, reports of having suffered suchinjuries are twice as frequent as they are in countries where whiplash is notcovered.  Presently,no objective test for whiplash exists, so it is true that spurious reports ofwhiplash injuries cannot be readily identified.  Nevertheless, these facts do notwarrant a conclusion that has been drawn by some commentators:  that in the countries with the higher ratesof reported whiplash injuries, half of the reported cases are spurious.  Clearly, in countries where automobileinsurance does not include compensation for whiplash, people often have littleincentive to report whiplash injuries that they actually have suffered.

Inthe argument given, the two boldfaced portions play which of the followingroles?
A.    The first is a claim that the argument disputes; thesecond is a conclusion that has been based on that claim.
B.     The first is claim that has been used to support aposition that the argument accepts; the second is a position that the argumentrejects.
C.    The first is a finding whose accuracy is evaluated in the argument; the second is the judgment reached by the argument concerning the accuracy of the finding.
D.    The first is a finding whose implications are at issue in the argument; the secondis the judgment reached by the argument concerning one alleged implication.
E.     The first is a finding, the explanation of which is at issue in the argument; thesecond is an objection that has been raised against theexplanation that the argument defends.   
OA: D

PREP08_cr 115.        (24536-!-item-!-188;#058&000941)
In countries where automobile insurance includes compensation for whiplash injuries sustained in automobile accidents, reports of having suffered such injuries are twice as frequent as they are in countries where whiplash is not covered. Some commentators have argued, correctly, that since there is presently no objective test for whiplash,spurious reports of whiplash injuries cannot be readily identified. These commentators are, however, wrong to draw the further conclusion that in the countries with the higher rates of reported whiplash injuries, half of the reported cases are spurious:  clearly, in countries where automobile insurance does not include compensation for whiplash, people often have little incentive to report whiplash injuries that they actually have suffered.
In the argument given, the two boldfaced portions play which of the following roles?

A. The first is evidence that has been used to support a conclusion that the argument criticizes; the second is that conclusion.
B. The first is evidence that has been used to support a conclusion that the argument criticizes; the second is the position that the argument defends.
C. The first is a claim that has been used to support a conclusion that the argument accepts; the second is the position that the argument defends.
D. The first is an intermediate conclusion that has been used to support a conclusion that the argument defends; the second is the position that the argument opposes.
E. The first presents a claim that is disputed in the argument; the second is a conclusion that has been drawn on the basis of that claim.

OA: A


OG12_78

In countries where automobile insurance includes compensation for whiplash injuries sustained in automobile accidents, reports of having suffered such injuries are twice as frequent as they are in countries where whiplash is not covered. Presently, no objective test for whiplash exists, so it is true that spurious reports of whiplash injuries cannot be readily identified. Nevertheless, these facts do not warrant the conclusion drawn by some commentators, that in the countries with the higher rates of reported whiplash injuries, half of the reported cases are spurious. Clearly, in countries where automobile insurance does not include compensation for whiplash, people often have little incentive to report whiplash injuries that they actually have suffered.

In the argument given, the two boldfaced portions play which of the following roles?

A,The first is a claim that the argument disputes; the second is a conclusion that has been based on that claim.  
B,The first is a claim that has been used to support a conclusion that the argument accepts; the second is that conclusion.  
C,The first is evidence that has been used to support a conclusion for which the argument provides further evidence; the second is the main conclusion of the argument.  
D,The first is a finding whose implications are at issue in the argument; the second is a claim presented in order to argue against deriving certain implications from that finding.  
E,The first is a finding whose accuracy is evaluated in the argument; the second is evidence presented to establish that the finding is accurate.

OA: D

In countries where automobile insurance includes compensation for whiplash injuries sustained in automobile accidents, reports of having suffered such injuries are twice as frequent as they are in countries where whiplash is not covered.  
=>fact/background: about report on injures  
Some commentators have argued, correctly, that since there is presently no objective test for whiplash, spurious reports of whiplash injuries cannot be readily identified.  
=>commentator's view: the repots is spurious
These commentators are, however, wrong to draw the further conclusion that in the countries with the higher rates of reported whiplash injuries, half of the reported cases are spurious:  
=>author's view directly refute the main conlusion( not straightly stated in passage)
clearly, in countries where automobile insurance does not include compensation for whiplash, people often have little incentive to report whiplash injuries that they actually have suffered.
=>support the author's main conclusion

哦 我觉得这个题的逻辑链是这样的:先是给出一个事实(在有xx伤赔偿的地方,XX伤赔偿的报告要比没有XX伤赔偿的地方多出一倍),然后根据这个事实+另外一个事实(没有对这种伤的客观评价)一些人得出一个结论:一些人伪造报告很难识别。然后作者发表了一个声明(这些人得出的进一步的结论说一半的报告都是伪造的是错误的),然后说出了自己的一个理由(在那些没有赔偿的地方,人们很少报告自己受伤)。

http://forum.chasedream.com/thread-508615-1-1.html 6#
28,This facts是指上面提到的1.摩托车保险中包含whiplash injuries的国家A的此类受伤报告是没有包含的国家B的两倍2.实际上现在并不存在可以检测whiplash的方法,因此受伤报告难以鉴别。Nevertheless转折,下面的黑体部分对评论家根据上述facts得出的结论one implication产生质疑,结论说A国的报告有一半都是假的,B国的人就算受伤了也不上报。总的来说就是作者反对facts的推论,但没有对facts的真实性提出质疑。C错,选D


作者: lululuna    时间: 2015-7-19 16:06
7/19
either-or-型错误
非A即B=》A+B效果加成

GWD2-27(错题)
Rye sown in the fall and plowed into the soil in early spring leaves a residue that is highly effective at controlling broad-leaved weeds, but unfortunately for only about forty-five days. No major agricultural crop matures from seed in as little as forty-five days. Synthetic herbicides, on the other hand, although not any longer-lasting, can be reapplied as the crop grows. Clearly, therefore, for major agricultural crops, plowing rye into the soil can play no part in effective weed control.

The argument is most vulnerable to the objection that it fails to
(A) consider that there might be minor, quick-growing crops that do mature in forty-fi ve days or less
(B) identify any alternative method of weed control that could be used instead of the method it rejects
(C) distinguish among the various kinds of synthetic herbicides
(D) allow for the possibility of combining the two weed-control methods it mentions
(E) allow for the possibility that plants other than rye, handled the same way, might have the same effect

OA:  D
BG: rye sown in previous fall, leaves residue to control weeds (higly effective)
      synthetci herbicides can be reapplied althoug same effective duration
P:rye effective duration< 45days, but main agricultrual crops  >=45days to mature
C:  plowing rye can play no part in effective weed contrel

做一个类比
A 在除草非常有效,但是有效时间太短,而且得提前半年使用; =>A 的有优点 非常有效
B 虽然不比A时间长,但是可以在种植物时再次使用 => B的优点 再次使用

那么,用A的话完全不能有效控制
错=》A和B一起用更好, A前期highly effective,种下去之后用B(可以reapplied),完美结合两者优点。

联想:OG13_101
Which of the following most logically completes the argument?

The irradiation of food kills bacteria and thus retards spoilage. However, it also lowers the nutritional value of many foods. For example, irradiation destroys a significant percentage of whatever vitamin Bl a food may contain. Proponents of irradiation point out that irradiation is no worse in this respect than cooking. However, this act is either beside the point, since much irradiated food is eaten raw, or else misleading, since

(A) many of the proponents of irradiation are food distributors who gain from foods' having a longer shelf life
(B) it is clear that killing bacteria that may be present on food is not the only effect that irradiation has
(C) cooking is usually the final step in preparing food for consumption, whereas irradiation serves to ensure alonger shelf life for perishable foods
(D) certain kinds of cooking are, in fact, even more destructive of vitamin Bl than carefully controlledirradiation is
(E) for food that is both irradiated and cooked, the reduction of vitamin Bl associated with either process individually is compounded

OA: E
P: IRRADIATION owers food value no worse than COOKING
C: the argument is misleading

Irradiation lowers food value
Cooking lowers food value
=> IRRADIATION+COOKING lowers the food value further
IRRADIATION不好,COOKING也不好,两者加起来对食物更不好

已给出论据部分:
  1)因为Irradiated食物都是生吃的,那么cooking并不影响;
  2)对于要cooking的食物来说,cooking + irradiation 使得事物的营养变得更糟糕。


OG EXPLAINATION:
By stating that irradiation destroys no more Bl than cookingdoes, the proponent seems to be suggesting that anyfood that is going to be cooked mightas well be irradiated because it
will end up with the same amount of Bl eitherway. But if the effects of radiation and cooking combine to destroy more Bl than cooking or irradiation alone would, then theproponents' claim suggests somethingthat is false.



作者: lululuna    时间: 2015-7-19 22:12
7/19错题
TN22_Q22:
The sense of delayedgratification, of working now for later pleasure,has helped shape the economic behavior of our society. However, that sense isno longer nurtured as consistently in our children as it once was. For example,it used to take a bit of patience to put together the toys that children got incereal boxes; now the toys come from the boxes whole.
Which of the followingis an assumption of the passage above?
A.    Thetoys in cereal boxes have changed partly because the economic conditions of oursociety have improved.
B.     Theinfluence of promotion gimmicks on the economic behavior of our society hasincreased over the years.
C.    Thetoys that used to come in cereal boxes were put together by the same childrenwho played with them.
D.    !Part of the pleasure of any toylie in putting the toy together before playing with it.
E.     Today'schildren do not expect a single toy to provide pleasure for a long period oftime.

OA: C
P: now= child take whole box; previous=  take patience to put pieces together
C:   child is no longer nurtured the delayed gratification( working first to gain pleasure later)

assumption: child put the pieces together

作者: lululuna    时间: 2015-7-19 23:24
Q31: FF-19.
Jane: Television programs and movies thatdepict violence among teenagers are extremely popular. Given how influentialthese media are, we have good reason to believe that these depictions causeyoung people to engage in violent behavior. Hence, depictions of violence amongteenagers should be prohibited from movies and television programs, if only inthose programs and movies promoted to young audiences.
Maurice: But you are recommending nothingshort of censorship! Besides which, your claim that television and moviedepictions of violence cause violence is mistaken: violence among young peoplepredates movies and television by centuries.
Which one of the following, if true, moststrengthens Jane’s argument?
A.   Themost violent characters depicted in movies and on television programs are adultcharacters which are portrayed by adult actors.
B.     Themovies that have been shown to have the most influence on young people’sbehavior are those that are promoted to young audiences.
C.   Thepeople who make the most profits in the movie and television industry are thosewho can successfully promote their work to both young and old audiences.
D.   Manyadolescents who engage in violent behavior had already displayed such behaviorbefore they were exposed violence in movies.
E.     Among theproducers who make both movies and television programs, many voluntarilyrestrict the subject matter of films directed toward young audiences.

OA: B
P:violence depictions cause young people to engage in violent behavior
C: in those programs and movies promoted to young audiences , depictions of violence among teenagers should be prohibited


Q32:GWD-13-Q5
Vorland’s government is planning a nationwide ban on smoking inrestaurants.  The objection that the banwould reduce restaurants’ revenues is ill founded.  Several towns in Vorland enacted restaurantsmoking restrictions five years ago. Since then, the amount the government collects in restaurant meal taxesin those towns has increased 34 percent, on average, but only 26 percentelsewhere in Vorland.  The amountcollected in restaurant meal taxes closely reflects restaurants’ revenues.
Which of the following, if true, most undermines the defense of thegovernment’s plan?
A.   When the state firstimposed a restaurant meal tax, opponents predicted that restaurants’ revenueswould decline as a result, a prediction that proved to be correct in the shortterm.
B.    The tax on meals in restaurants ishigher than the tax on many other goods and services.
C.   Over the last fiveyears, smoking has steadily declined throughout Vorland.
D.   In many of the townsthat restrict smoking in restaurants, restaurants can maintain separate diningareas where smoking is permitted.
E.   Over the last fiveyears, government revenues from sales taxes have grown no faster in the townswith restaurant smoking restrictions than in the towns that have no suchrestrictions.

OA:  D
P:  restaurant that enacted smoking restrictions got 34% increase in meal taxes, while others only 26%
C:  ban on smoking not reduce meal revenue




作者: lululuna    时间: 2015-7-20 20:37
7/20
同源题
GWD30-Q38:
Theancient Nubians inhabited an area in which typhus occurred, yet surprisinglyfew of their skeletons show the usual evidence of this disease.  The skeletons do show deposits oftetracycline, an antibiotic produced by a bacterium common in Nubian soil.  This bacterium can flourish on the dried grainused for making two staples of the Nubian diet, beer and bread.  Thus, tetracycline in their food probablyexplains the low incidence of typhus among ancient Nubians.
P: Nubian skeletons no evidence of typhus(disease) but deposits of tetracycline(antibiotic)
C: food ( tetracycline fluorished on dried grain the staple of Nubian) explains the low incidence

gap:  deposits of bactria ≠ bacteria in food=》 possibly from other resoures

Which of the following is an assumption on which the argumentrelies?
A.    The tetracycline deposits did not form after the bodies were buried.
B.     The diseases other than typhus to which the ancient Nubians wereexposed would not be affected by tetracycline.
C.    Typhus is generally fatal.
D.    Nubian grain became contaminated with tetracycline-producingbacteria prior to being harvested.
E.     Bread and beer were the only foods eaten by the ancient Nubianswhich could have contained tetracycline.
oa: A

GWD18-Q30
The ancient Nubians inhabited anarea in which typhus occurs, yet surprisingly few of their skeletons show theusual evidence of this disease.  Theskeletons do show deposits of tetracycline, an antibiotic produced by abacterium common in Nubian soil.  Thisbacterium can flourish on the dried grain used for making two staples of theNubian diet, beer and bread.  Thus,tetracycline in their food probably explains the low incidence of typhus amongancient Nubians.
Which of the following is an assumption on whichthe argument relies?
A.   Infectiousdiseases other than typhus to which the ancient Nubians were exposed areunaffected by tetracycline.
B.    Tetracycline is not rendered ineffective as anantibiotic by exposure to the processes involved in making bread and beer.
C.   Typhuscannot be transmitted by ingesting bread or beer contaminated with theinfectious agents of this disease.
D.   Breadand beer were the only items in the diet of the ancient Nubians which couldhave contained tetracycline.
E.    Typhusis generally fatal.

OA: B
P: Nubian skeletons no evidence of typhus(disease) but deposits of tetracycline(antibiotic)
C: food ( tetracycline fluorished on dried grain for the staple of Nubian) explains the low incidence

gap:  deposits of bactria ≠ bacteria in food=》 possibly from other resoures


作者: lululuna    时间: 2015-7-20 21:15
7/20错题


羊皮狼GWD-29-Q38
In the past, every ten-percentage-point increase in cigaretteprices in the country of Coponia has decreasedper capita sales of cigarettes by four percent.  Coponia is about to raise taxes on cigarettesby 9 cents per pack.  The average price of cigarettes in Coponia isand has been for more than a year 90 cents perpack.  So the tax hike stands anexcellent chance of reducing per capita sales of cigarettes by four percent.

BG: cig price increases 10%, then per captia sale decreases 4%;
P: raise tax on cig by 9 cents, and the avg prc this year has been 90 cents;
C:  reduce the per captia cig by 10%

gap/intermediate conclusion:  cig prc will increased by 10%( 9/90)
Which of the following is an assumption on which the argument depends?
A.    To baccocompanies are unlikely to reduce their profit per pack of cigarettes to avoidan increase in the cost per pack to consumers in Coponia.
B.     Previous increases in cigarette prices in Coponia have generally been due to increasesin taxes on cigarettes.
C.    Any decrease in per capita sales of cigarettes in Coponia will result mainly froman increase in the number of people who quit smoking entirely.
D.    At present, the price of a pack of cigarettes in Coponia includes taxes thatamount to less than ten percent of the total selling price.
E.     The number of people in Coponia who smoke cigarettes has remained relativelyconstant for the past several years.
OA: A


一点自醒的tips:
这题做的时间长,还做错了,一直纠结在per captia上面,但其实不是计算型的逻辑题;
因为per captia已经在background info 里面作为general rule给出了,是已知条件,不是assumption,不需要去根据它来判断。
是断桥型的题目,只给出了tax增加9cents,之前价格是90cents=>implied的结果是 价格增长了10%,但却没有直接说出,留下了逻辑漏洞,此处是破题点



GWD30-Q32:
A major chemical spill occurred five years ago atBaker’s Beach, the world’s sole nesting ground for Merrick sea turtles, andprevented nearly all the eggs laid that year from hatching.  Yet the number of adult female Merricks returningto lay their eggs at Baker’s Beach has actually increased somewhat since fiveyears ago.  Clearly, environmentalists’prediction that the world’s Merrick population would decline as a result of thespill has proven unfounded.
Whichof the following, if true, most seriously underminesthe argument offered in refutation of theenvironmentalists’ prediction?
A.    The chemical spill five years ago occurred at a time when there were neitherMerrick sea turtles nor Merrick sea turtle eggs on Baker’s Beach.
B.     Female Merrick sea turtles begin returning to Baker’s Beach to lay their eggs whenthey are ten years old.
C.    Under normal conditions, only a small proportion of hatchling female Merrick seaturtles survive in the ocean until adulthood and return to lay their eggs atBaker’s Beach.
D.    Environmental pressures unrelated to the chemical spill have caused a significant decline inthe population of one of the several species of sea birds that prey on Merricksea turtle eggs.
E.     After the chemical spill, an environmental group rejected a proposal to increase theMerrick sea turtle population by transferring eggs from Baker’s Beach to nearbybeaches that had not been affected by the spill.

Which of the following, if true, most seriously undermines the argument offered in refutation of the environmentalists’ prediction?
P: spill occurred 5yrs ago and prevented nearly all the eggs laid that year from hatching, but # female M returnning to lay eggs has increased 5 yrs ago;
C:   E's prediction that M # would decline is unfouned = in refutation of the environmentalists’ prediction

=>thus stengthen the E's predition is what we should do.

B 、Female M是那些已经长到10岁的龟,但是spill是在5年前发生的,说明这批生蛋的母M 不是 5年前的出生母M=>从而不能说明 M#没有减少=>加强了M#减少的可能性





作者: lululuna    时间: 2015-7-21 20:15
7/21错题

GWD-13-Q32*:
Macrophagesare cells that play a role in the response of the immune system of mice andother mammals to invasive organisms such as bacteria.  Unlike other mice, mice that are geneticallyincapable of making these particular cells do not show elevated levels ofnitrates when infected with bacteria.

Thestatements above, if true, provide the most support for which of the followingconclusions?
A.   Mice that are unable either tomake macrophages or to make them in sufficient numbers will protect themselvesfrom bacterial infections in some other way.
B.    Mice that show elevated levelsof nitrates can easily fight off most types of bacterial infections.
C.   In mice, macrophages playa role in the production of nitrates or inhibit a process by which nitrates arebroken down or otherwise eliminated.
D.   When a healthy mouse becomesinfected with an invasive organism, the number of macrophages in the mouse’sbody decreases.
E.    Injections of nitrates intomice that lack macrophages will not enhance the ability of these animals’immune systems to fight off infection.

OA:C
M protect mice from bacteria;
if mice can’t produce M,they didn't show increased level of N when infected with bacteria;
=>whether N increase or not is related to the prodcution of M
=>正确答案必须指向M和N之间的关系

TN23_Q35:
GWD-11-Q31:
People with a certain eye disorder are virtually unable to see inmoderately bright light, which seems to them unbearably intense, since thecells of their retinas are overwhelmed by moderately bright light.  These people do, however, show normal sensitivity to most componentsof dim light.  Their retinal cells arealso not excessively sensitive to red components of moderately bright light.
The information above best supports which of the followinghypotheses about people with the disorder described, if they have no otherserious visual problems?

A.   In all moderately dimlight in which people without the disorder can read large print, people withthe disorder cannot read such print.
B.    In an otherwisedarkened concert hall, these people will see a dimly illuminated red exit signmore clearly than small dim white lights that mark the aisles.
C.   These peopletypically see more acutely at night and in dim light than do most people who donot have the disorder.
D.   Eyeglasses that aretransparent to red components of light but filter out other components of lighthelp these people see in moderately bright light.
E.    These people perceive colors other than red in the same way as domost people who do not have the disorder.

OA : D

F1:can't see bright light; but can see dim light;
F2:not excessively sensitive to red of bright light;

GWD23_Q36: OV-80
Vitacorp, amanufacturer, wishes to make its information booth at an industry conventionmore productive in terms of boosting sales. The booth offers informationintroducing the company’s new products and services. To achieve the desiredresult, Vitacorp’s marketing department will attempt to attract more people tothe boost. The marketing director’s first measure was to instruct eachsalesperson to call his or her five best customers and personally invite themto visit the boost.
Which of thefollowing, if true, most seriously weaken the predication that the marketingdirector’s first measure will contribute to meeting that goal of boostingsales?
A.   Customers who participate booth are thosenot satisfied with Vitacorp products and will use this opportunity toexpress their unsatisfaction.
B.    Many of Vitacorp’s competitors have made plans formaking their own information booths more productive in increasing sales.
C.   An information booth that is well attended tends toattract visitors who would not otherwise have attended the booth.
D.   Most of Vitacorp’s best customers also havebusiness dealings with Vitacorp’s competitors.
E.    Vitacorp has fewer new products and servicesavailable this year than it had in previous years.

oa: A

OG-199 http://forum.chasedream.com/thread-14936-1-1.html
199.
Vitacorp, a manufacturer, wishes to make its information booth at an industry convention more
productive in terms of boosting sales. The booth offers information introducing the company’s
new products and services. To achieve the desired result, Vitacorp’s marketing department will
attempt to attract more people to the both. The marketing director’s first measure was to
instruct each salesperson to call his or her five best customers and personally invite them to
visit the booth.
Which of the following, if true, most strongly supports the prediction that the marketing
director’s first measure will contribute to meeting the goals of boosting sales?
(A) Vitacorp’s salespeople routinely inform each important customer about new products and
services as soon as the decision to launch them has been made.
(B) Many of Vitacorp’s competitors have made plans for making their won information booths
more productive in increasing sales.
(C) An information booth that is well attended tends to attract visitors who would not otherwise
have attended the booth.
(D) Most of Vitacorp’s best customers also have business dealings with Vitacorp’s
competitors.
(E) Vitacorp has fewer new products and services available this year than it had in previous
years.
OA:c


作者: lululuna    时间: 2015-7-22 16:06
同源题

OG12
Lyme disease is caused by a bacterium transmitted to humans by deer ticks. Generally, deer ticks pick up the bacterium while in the larval stage by feeding on infected white-footed mice. However, certain other species on which the larvae feed do not harbor the bacterium. If the population of these other species were increased, more of the larvae would be feeding on uninfected hosts, so the number of ticks acquiring the bacterium would likely decline.

Which of the following would it be most important to ascertain in evaluating the argument?
A,Whether populations of the other species on which deer tick larvae feed are found only in areas also inhabited by white-footed mice   
B,Whether the size of the deer tick population is currently limited by the availability of animals for the tick's larval stage to feed on
C,Whether the infected deer tick population could be controlled by increasing the number of animals that prey on white-footed mice   
D,Whether deer ticks that were not infected as larvae can become infected as adults by feeding on deer on which infected deer ticks have fed   
E,Whether the other species on which deer tick larvae feed harbor any other bacteria that ticks transmit to humans
OA:B

T-9-Q28
Lyme disease is caused by a bacterium transmitted to humans by deer ticks. Generally deer ticks pick up the bacterium while in the larval stage from feeding on infected white-footed mice. However, certain other species on which the larvae feed do not harbor the bacterium. Therefore, if the population of these other species were increased, the number of ticks acquiring the bacterium and hence the number of people contracting Lyme disease would likely decline.
Which of the following, if true, most strengthens the argument?
A.    Ticks do not suffer any adverse consequences from carrying the bacterium that causes Lyme disease in humans.
B.    There are no known cases of a human’s contracting Lyme disease through contact with white-footed mice.
C.    A deer tick feeds only once while in the larval stage.
D.    A single host animal can be the source of bacterium for many tick larvae.
E.     None of the other species on which deer tick larvae feed harbor other bacteria that ticks transmit to humans
OA:C

BG: L is caused bacteria by deer tick ;
      tick pick b at larval stage

P: # other species(food of tick at larva stage) increases
inter C: # tick acquring bacteria will decline
C: # people contracting L will decline
作者: lululuna    时间: 2015-7-22 22:18
7月22日错题

T-9-Q3
Business Consultant: some corporations shun the use of executivetitles because they fear that the use of titles indicating position in thecorporation tends to inhibit communication up and down the corporate hierarchy.Since an executive who uses a title is treated with more respect by outsiders, however, use of a title canfacilitate an executive’s dealings with external businesses. The obviouscompromise is for these executives to use their corporate titles externally butnot internally, since even if it is widely known that the corporation’sexecutives use executive titles outside their organization, this knowledge doesnot by itself inhibit communication within the corporation.

In the consultant’s reasoning, the two portions in boldface playwhich of the following roles?
A.    The first presents an obstacle to achieving a certain goal; thesecond presents a reason for considering that goal to be undesirable.
B.     The first is a consideration that has led to the adoption of acertain strategy; the second presents a reason against adopting that strategy.  
C.    The first describes a concern that the consultant dismisses asinsignificant; the second is a consideration that serves as the basis for that dismissal.
D.    The first is a belief for which the consultant offers support; the second is part of thatsupport.
E.     The first is a belief against which evidence is offered; the secondis part of the evidence offered against that belief.
OA : B

PREP08_85. (23981-!-item-!-188;#058&000656)
Fearing that the use of titles indicating positionin the corporation tends to make the corporate hierarchy rigid by inhibitingcommunication, some corporations shun the use of executive titles.  A title, however, can facilitate anexecutive's dealings with external businesses since it encourages outsiders totreat the executive with respect.  Theobvious compromise is for these executives to use their titles externally butnot within their corporations.


Which of the following, if true, provides the mostsupport for the compromise suggested above?
A. Only small corporations can preserve anatmosphere of mutual respect and high regard without having a rigid corporatehierarchy.
B. Referring to an executive by using a title canencourage both those outside the organization and inside the organization totreat the executive with respect.
C. Even if it is widely known within a corporationthat the corporation's executives use executive titles outside theirorganizations, this knowledge does not by itself inhibit communication withinthe corporation.
D. A rigid corporate hierarchy can promoteefficiency within an organization as well as provide access to the corporationfor those outside the organization.
E. Although many corporateexecutives disapprove of rigid hierarchies on the grounds that they inhibitcommunication, the vast majority of executives have no qualms about usingtitles both internally and externally.   
OA: C

BG:shun the use of title inside, because it makes hierarchy which inhibite communication
P:
[size=13.3333330154419px]facilitate an executive's dealings with external businesses  
C: use their titles externally but not within their corporations




T-9-Q4
In the years following an eight-cent increase in the federal tax ona pack of cigarettes, sales of cigarettes fell ten percent. In contrast, in theyear prior to the tax increase, sales had fallen one percent. The volume ofcigarette sales is therefore strongly related to the after-tax price of a packof cigarettes.
Theargument above requires which of the following assumptions?
A.    Duringthe year following the tax increase, the pretax price of a pack of cigarettesdid not increase by as much as it had during the year prior to the taxincrease.
B.    Theone percent fall in cigarette sales in the year prior to tax increases was dueto a smaller tax increase.
C.    Thepretax price of a pack of cigarettes gradually decreased throughout the yearbefore and the year after the tax increase.
D.    Forthe year following the tax increase, the pretax price of a pack of cigaretteswas not eight or more cents lower than it had been the previous year.
E.     Asthe after-tax price of a pack of cigarettes rises, the pretax price also rises.
OA:  d

P: increase tax, sale fell
C: after-tax prc relate to cig Sale
gap :brideg tax to prc
提高tax以后pretax price不会比前年低的超过8 cents,比如前年10 cents,今年2 cents。那么tax上升了8 centsprice下降的少于8 cents,所以after-tax price还是涨价了,sales下降。加强。如果取非,那就是说今年pretax price降低的比提高的tax还高,那么cigarette价格其实是降低了。Sales应该升高才对,而原文是sales降低。

同源题

In the year following an eight-cent increase in the federal tax on a pack of cigarettes, sales of cigarettes fell ten percent. In contrast, in the year prior to the tax increase, sales had fallen one percent. The volume of cigarette sales is therefore strongly related to the after-tax price of a pack of cigarettes.

Which of the following, if true, would most strengthen the argument above?

A,During the second year after the tax increase, cigarette sales increased by a significant amount.
B,The information available to consumers on the health risks of smoking remained largely unchanged in the period before and after the tax increase.  
C,Most consumers were unaware that the tax on cigarettes was going to increase.  
D,During the year following the cigarette tax increase, many consumers had less income, in inflation-adjusted dollars, than they had had in the previous year
E,During the year after the tax increase, there was a greater variety of cigarettes on the market than there had been during the previous year.  

OA: B
作者: lululuna    时间: 2015-7-23 12:58
同源题

GMAT-PREP08-CR1
Which of the following most logically completes the passage?
Concerned about the financial well-being of its elderly citizens, the government of Runagia decided two years ago to increase by 20 percent the government-provided pension paid to all Runagians over 65.  Inflation in the intervening period has been negligible, and the increase has been duly received by all eligible Runagians.  Nevertheless, many of them are no better off financially than they were before the increase, in large part because __________.
A. they rely entirely on the government pension for their income
B. Runagian banks are so inefficient that it can take up to three weeks to cash a pension check
C. they buy goods whose prices tend to rise especially fast in times of inflation
D. the pension was increased when the number of elderly Runagians below the poverty level reached an all-time high
E. in Runagia children typically supplement the income of elderly parents, but only by enough to provide them with a comfortable living
OA: E

OG13_24
Plan: Concerned about the welfare of its senior citizens, the government of Runagia decided two years ago to increase by 20 percent the government-provided pension paid to all Runagians age sixty-five and older.
Result: Many Runagian senior citizens are no better off financially now than they were before the increase.
Further information: The annual rate of inflation since the pension increase has been below 5 percent, and the increased pension has been duly received by all eligible Runagians.

In light of the further information, which of the following, if true, does most to explain the result that followed implementation of the plan?
(A) The majority of senior citizens whose financial position has not improved rely entirely on the government pension for their income.
(B) The Runagian banking system is so inefficient that cashing a pension check can take as much as three weeks.
(C) The prices of goods and services that meet the special needs of many senior citizens have increased at a rate much higher than the rate of inflation.
(D) The pension increase occurred at a time when the number of Runagians age sixty-five and older who were living below the poverty level was at an all-time high.
(E) The most recent pension increase was only the second such increase in the last ten years.
OA: C

prep08_87. (31284-!-item-!-188;#058&005747)
Two years ago, the government of Runagiaincreased by 20 percent the government-provided pensions paid to Runagians over65.  The aim of the increase was tostimulate the economy in the rural regions of the country, where most pensionrecipients live.  Statistics, however,show that there has been no increase in economic activity in those regionssince then, but that there has been noticeably more spending in the urbanareas.

Which of the following, if true, most helpsto explain why the increase resulted in the unintended state of affairsdescribed?
A. Until the pensions were increased, manyRunagians over 65 in rural regions had been receiving support from theirchildren who live in urban areas.
B. The pensions were increased when thenumber of people below the poverty level in rural areas of Runagia reached anall-time high.
C. City-dwellers in Runagia rarely travelto rural regions of the country.
D. The Runagian postal system is soinefficient that it can take up to three weeks for pension checks to reachrecipients in rural areas.
E. On average, the pensions were higher inrural than in urban areas before the increase.

OA:A



作者: lululuna    时间: 2015-7-26 22:51
7/26

T-4-Q2Twenty years ago, Balzania put in place regulations requiring operators of surface mines to pay for the reclamation of mined-out land.Since then, reclamation technology has not improved.Yet, the average reclamation cost for a surface coal mine being reclaimed today is only four dollars per ton of coal that the mine produced, less than half what it cost to reclaim surface mines in the years immediately after the regulations took effect.

Which of the following, if true, most helps to account for the drop in reclamation costs described?

A. Even after Balzania began requiring surface mine operators to pay reclamation costs, coal mines in Balzania continued to be less expensive to operate than coal mines in almost any other country.
B. In the twenty years since the regulations took effect, the use of coal as a fuel has declined from the level it was at in the previous twenty years.
C. Mine operators have generally ceased surface mining in the mountainous areas of Balzania because reclamation costs per ton of coal produced are particularly high for mines in such areas.
D. Even after Balzania began requiring surface mine operators to pay reclamation costs, surface mines continued to produce coal at a lower total cost than underground mines.
E. As compared to twenty years ago, a greater percentage of the coal mined in Balzania today comes from surface mines.


OA: C

作者: lululuna    时间: 2015-7-27 00:22
Prep2012-Pack1-CR-071       VCR014016     Hard
Electric utilities pay less for low-quality coal per ton delivered than for high-quality coal. Yet more low-quality coal   than high—quality coal must be burned to generate the same amount of electricity. Moreover, per ton of coal burned, low-quality coal generates more ash than does high-quality coal, and the disposal of ash is becoming more and more expensive.
The considerations above, if true, most strongly support which of the following claims?
A.   A coal-burning utility might not be assured of benefiting economically by always adhering to the policy of keeping its overall coal purchasing costs as low as possible.
B.  In those regions where the cost of disposing of coal ash is negligible, it is more expensive for coal-burning utilities to use high-quality coal than low-quality coal.
C. Transportation costs represent a smaller proportion of the cost per delivered ton for low-quality coal than for high-quality coal.
D. It is no less expensive to dispose of a ton of coal ash that results from the burning of high-quality coal than it is to dispose of a ton of coal ash that results from the burning of low-quality coal.
E. In regions where coal-ash disposal is the least expensive, reserves of low-quality coal are likely to decline at a faster rate than are reserves of high-quality coal.
OA:e
FACT1: @purchasing prc: low-q<h-q
FACT2: @produce same # of electiric : # low-q> #h-q
Fact3:  @# ash/ ton burned: low-q>  h-q  while the disposal fee of ash is increasing


作者: lululuna    时间: 2015-7-27 00:35
同原题

GWD3_30
The percentage of households with an annual income of more than $40,000 is higher in Merton county than in any other county.However, the percentage of households with an annual income of $60,000 or more is highest in Sommer county.

If the statements above are true, which of the following can properly be concluded on the basis of them

  A,No household in Merton county has an annual income of $60,000 or more
  B, Some households in Merton county have an annual income between $40,000 and $60,000.
  C, The number of households with an annual income of more than $40,000 is greater in Merton than in Sommer county
  D, Average annual household income is higher in Sommer than in Merton county.
  E, The percentage of households with an annual income of $80,000 is higher in Sommer than in Merton county.

OA:B

Prep2012-Pack1-CR-065        VCR003842        HardThe percentage of households with an annual income of more than $40,000 is higher in Merton County than in any other county. However, the percentage of households with an annual income of $60,000 or more is higher in Sommer County.

If the statements above are true, which of the following must also be true?
A.        The percentage of households with an annual income of $80,000 is higher in Sommer County than in Merton County.
B.        Merton County has the second-highest percentage of households with an annual income of $60,000 or more.
C.        The percentage of households with an annual income between $40,000 and $60,000 is higher in Merton County than in Sommer County.
D.        The total number of households with an annual income of more than $40,000 is greater in Merton County than in Sommer County.
E.        Average annual household income is higher in Sommer County than in Merton County.

OA: C


作者: lululuna    时间: 2015-7-30 19:20
7/30

OG分册Verbal5.
In an attempt to promote the widespreaduse of paper rather than plastic, and thus reduce nonbiodegradable waste, thecouncil of a small town plans to banthe sale of disposable plastic goods for whichsubstitutes made of paperexist. The council argues that since most paper is entirely biodegradable, paper goods are environmentally preferable.

Which of the following, if true, indicates that the planto ban the sale of disposable plastic goods is ill suited to the town council’s environmental  goals?

(A)       Althoughbiodegradable plastic goods are now available, members of the town council believe biodegradable paper goods to be safer for the environment.
(B)       Thepaper factory at which most of the townspeople are employed plans to increase production of biodegradable paper goods.
(C)       Afterother towns enacted similar bans on the sale of plastic goods, the environmentalbenefits were not discernible for several years.
(D)       Sincemost townspeople prefer plastic goodsto paper goods in many instances,theyare likely to purchasethem in neighboring towns where plastic goods are available for sale.
(E)       Productsother than those derived from wood pulpare often used in the manufacture of paper goods that are entirelybiodegradable.

P:plans to ban the sale of disposable plastic goods
C:to promote the widespread use of paper,and thus reduce nonbiodegradable waste


OA:  D

The town council’s plan is, however, only as effective as the support it has from the town’s citizens. If the citizens prefer disposable plastic goods and if the goods are also readily available in neighboring towns, there is nothing to stop them from buying the plastic goods elsewhere if that is their preference.


作者: theonefromhell    时间: 2015-9-15 05:40
顶楼主!
<a id="fasterR3" href="javascript:;">感谢分享!</a>
<span style="line-height: 25px;">快速回复:</span>
<script type="text/javascript">
if(getcookie('fastpostrefresh') == 1) {$('fastpostrefresh').checked=true;}
$('fasterR1').onclick = $('fasterR2').onclick = $('fasterR3').onclick =$('fasterR4').onclick = function(){
fasterRSub(this);
}
function fasterRSub(obj){
$('fastpostmessage').innerHTML = obj.innerHTML + '               ';
$('fastpostsubmit').click();
}
</script>
               
作者: lululuna    时间: 2015-10-9 15:49
http://forum.chasedream.com/thread-563491-1-1.html

http://forum.chasedream.com/thread-564066-1-1.html

http://forum.chasedream.com/thread-565843-1-1.html

http://forum.chasedream.com/thread-565843-1-1.html

http://forum.chasedream.com/thread-567796-1-1.html

http://forum.chasedream.com/thread-568790-1-1.html

http://forum.chasedream.com/thread-568790-1-1.html

http://forum.chasedream.com/thread-569879-1-1.html

http://forum.chasedream.com/thread-570678-1-1.html

http://forum.chasedream.com/thread-572177-1-1.html

http://forum.chasedream.com/thread-572711-1-1.html

http://forum.chasedream.com/thread-573332-1-1.html

http://forum.chasedream.com/thread-574330-1-1.html

http://forum.chasedream.com/thread-576100-1-1.html

http://forum.chasedream.com/thread-583286-1-1.html

http://forum.chasedream.com/thread-593839-1-1.html
http://forum.chasedream.com/thread-626665-1-1.html

作者: miss不忘初心    时间: 2016-1-18 00:45
In countries where automobile insurance includes compensation for whiplash injuries sustained in automobile accidents, reports of having suffered such injuries are twice as frequent as they are in countries where whiplash is not covered. Some commentators have argued, correctly, that since there is presently no objective test for whiplash, spurious reports of whiplash injuries cannot be readily identified. These commentators are, however, wrong to draw the further conclusion that in the countries with the higher rates of reported whiplash injuries, half of the reported cases are spurious: clearly, in countries where automobile insurance does not include compensation for whiplash, people often have little incentive to report whiplash injuries that they actually have suffered.

In the argument given, the two boldfaced portions play which of the following roles?
AThe first is evidence that has been used to support a conclusion that the argument criticizes; the second is that conclusion.  分析A选项
BThe first is evidence that has been used to support a conclusion that the argument criticizes; the second is the position that the argument defends.  分析B选项
CThe first is a claim that has been used to support a conclusion that the argument accepts; the second is the position that the argument defends.  分析C选项
DThe first is an intermediate conclusion that has been used to support a conclusion that the argument defends; the second is the position that the argument opposes.  分析D选项
EThe first presents a claim that is disputed in the argument; the second is a conclusion that has been drawn on the basis of that claim.  分析E选项
作者: miss不忘初心    时间: 2016-1-18 00:46
LZ看到你之前有发过这题类似的变体,这题的话应该选择什么答案,能看看嘛?




欢迎光临 ChaseDream (https://forum.chasedream.com/) Powered by Discuz! X3.3